LSAT and Law School Admissions Forum

Get expert LSAT preparation and law school admissions advice from PowerScore Test Preparation.

 Administrator
PowerScore Staff
  • PowerScore Staff
  • Posts: 8916
  • Joined: Feb 02, 2011
|
#66070
Please post your questions below!
 lsat12345
  • Posts: 11
  • Joined: Jun 29, 2019
|
#66263
I posted on here before about this question, but I still don't feel confident about it. Just because Lycopene is more abundant in fruits and vegetables, doesn't mean that the people in the study got their lycopene dose from fruits and vegetables. The stimulus never states that the people in the study got their lycopene doses from fruits/veggies, so how can we make that leap in answer choice A?
 Malila Robinson
PowerScore Staff
  • PowerScore Staff
  • Posts: 296
  • Joined: Feb 01, 2018
|
#66437
Hi LSAT12345,
Since this is a Weaken question you can bring in outside information in order to highlight a problem in the argument. Answer A attacks the conclusion of the argument which says that lycopenes reduce the risk of stroke. Specifically, Answer A is saying that it may be other things that are reducing the risk of stroke, so the lycopenes may have nothing to do with it.
Hope that helps!
-Malila
 taylorharris24
  • Posts: 14
  • Joined: Aug 05, 2018
|
#66753
Malila Robinson wrote:Hi LSAT12345,
Since this is a Weaken question you can bring in outside information in order to highlight a problem in the argument. Answer A attacks the conclusion of the argument which says that lycopenes reduce the risk of stroke. Specifically, Answer A is saying that it may be other things that are reducing the risk of stroke, so the lycopenes may have nothing to do with it.
Hope that helps!
-Malila
How can we assume that any of the participants were eating the fruits and vegetables as mentioned before? All the info we have is that it is found in fruits and vegetables, we have no idea where the source of the participants lycopenes came from?
 James Finch
PowerScore Staff
  • PowerScore Staff
  • Posts: 943
  • Joined: Sep 06, 2017
|
#66759
Hi Taylor,

When we have a specific scientific reference for which there isn't any "common sense" outside knowledge that can be brought in, we have to use only the information in the stimulus as a guide. Here, we know we have a large study (1k people) over a 12-year period; the ones with more lycopene in their system are less likely to suffer a stroke. From that, we can make a likely inference that at least some of these thousand people are regularly eating the fruits and vegetables that are high in lycopene, as we know of no other way for a human to obtain it.

The main takeaway is to keep your eyes on the prize; this is a weaken question, with a causal conclusion, so we should be looking for answer choices that weaken causal conclusions: alternate causes, cause without effect (or vice versa), mere correlation, reverse causation. Here, (A) immediately give us an alternate cause, making it the correct answer choice.

Hope this clears things up!
User avatar
 lalalala
  • Posts: 13
  • Joined: Aug 04, 2023
|
#102664
Hi,

I understand from the previous replies that we can assume that people are eating the fruits and vegetables. However, I thought answer choice D was better than A because it actually tells us that people with high lyco were eating more fruits/vegetables, and provides an alternate cause––perhaps it's the other benefits in the fruits and vegetables, not necessarily the lyco, that made them less likely to have a stroke. I realize that's an assumption I'm making, but we also had to make an assumption in ac A, so I would just like some guidance on how to know when the assumption is warranted.

Thanks for all your help!
 Luke Haqq
PowerScore Staff
  • PowerScore Staff
  • Posts: 747
  • Joined: Apr 26, 2012
|
#102751
Hi lalala!

The argument in this stimulus is causal. The author is specifically arguing that "lycopene reduces the risk of stroke."

With that in mind, consider a difference between answer choice (A) versus (D). (D) is missing any mention of stroke, so it's unlikely to weaken/strengthen or do anything to an argument making a causal connection between lycopene and stroke risk. By contrast, both lycopene and strokes are discussed in answer choice (A). More specifically, (A) suggests an alternate cause, thus weakening the causal argument in the stimulus.
User avatar
 lalalala
  • Posts: 13
  • Joined: Aug 04, 2023
|
#103170
Luke Haqq wrote: Wed Aug 09, 2023 7:25 pm Hi lalala!

The argument in this stimulus is causal. The author is specifically arguing that "lycopene reduces the risk of stroke."

With that in mind, consider a difference between answer choice (A) versus (D). (D) is missing any mention of stroke, so it's unlikely to weaken/strengthen or do anything to an argument making a causal connection between lycopene and stroke risk. By contrast, both lycopene and strokes are discussed in answer choice (A). More specifically, (A) suggests an alternate cause, thus weakening the causal argument in the stimulus.
Thank you, Luke. That helps a lot! :)

Get the most out of your LSAT Prep Plus subscription.

Analyze and track your performance with our Testing and Analytics Package.